URGENT!! ILL GIVE
BRAINLIEST!!!! AND 100 POINTS!!!!!!

When a pole vaulter reaches the top of her vault, how does her potential energy compare to her kinetic energy?

A. The kinetic energy is twice the amount of potential energy.

B. The kinetic energy is greater than the potential energy.

C. The kinetic energy and potential energy are the same.

D. The kinetic energy is less than the potential energy.

Answers

Answer 1

Answer: b

Explanation:


Related Questions

what happens to F 1. m is doubled2. m is tripled3. m an m are both doubled4. m is halved5. r is doubled6. r is tripled7. r is increased 10 times8. r is halved

Answers

The expression for the gravitational force is given as:

[tex]F_g=\frac{GMm}{r^2}[/tex]

1. When M is doubled, then force will also be doubled as Force is directly proportional to the mass.

[tex]\begin{gathered} F\alpha M \\ F\alpha m \\ F\alpha\frac{1}{r^2} \end{gathered}[/tex]

2. When the m is tripled, then force will be tripled.

[tex]\begin{gathered} F^{\prime}\alpha\frac{M\times3m}{r^2} \\ F^{\prime}\alpha\frac{3Mm}{r^2} \\ F^{\prime}\alpha3F \end{gathered}[/tex]

3. When both M and m are doubled then, the force will become four times.

[tex]\begin{gathered} F^{\prime}\alpha\frac{2M\times2m}{r^2} \\ F^{\prime}\alpha4F \end{gathered}[/tex]

4.When m is halved, then Force will be halved, as force is directly proportional to the mass.

[tex]\begin{gathered} F^{\prime}\alpha\frac{M\times\frac{m}{3}}{r^2} \\ F^{\prime}\alpha\frac{Mm}{3r^2} \\ F^{\prime}\alpha\frac{F}{3} \end{gathered}[/tex]

5. When distance is doubled, then force will become one-fouth.

[tex]\begin{gathered} F^{\prime}\alpha\frac{Mm}{(2r)^2} \\ F^{\prime}\alpha\frac{Mm}{r^2}\times\frac{1}{4} \\ F^{\prime}\alpha\frac{F}{4} \end{gathered}[/tex]

6.When the distance is tripled, the force will become one-ninth.

[tex]\begin{gathered} F^{\prime}\alpha\frac{Mm}{(3r)^2} \\ F^{\prime}\alpha\frac{Mm}{r^2}\times\frac{1}{9} \\ F^{\prime}\alpha\frac{F}{9} \end{gathered}[/tex]

7. When the distance is increased to ten times, the force will become one-hundredth.

[tex]\begin{gathered} F^{\prime}\alpha\frac{Mm}{(10r)^2} \\ F^{\prime}\alpha\frac{Mm}{r^2}\times\frac{1}{100} \\ F^{\prime}\alpha\frac{F}{100} \\ 8.\text{ When the distance in halved, the force will be four time.} \\ F^{\prime}\alpha\frac{Mm}{(\frac{r}{2})^2} \\ F^{\prime}\alpha4\frac{Mm}{r^2} \\ F^{\prime}\alpha4F \end{gathered}[/tex]

A capacitor is constructed of two large, identical, parallel metal plates separated by a small distance d. A battery fully charges the capacitor and is then disconnected. If, instead of separating the plates, the empty space between the plates is filled with a slab of insulating material that has a dielectric constant κ= 2, which of the following correctly describes the change, if any, of the voltage across the capacitor, the electric field between the plates, and the energy stored in the capacitor? 1) Voltage halvesElectric field halves Energy halves 2. Voltage halves Electric field halves Energy does not change 3. Voltage does not change Electric field does not change Energy halves 4. Voltage does not changeElectric field halves Energy halves 5. Voltage does not change Electric field does not change Energy does not change

Answers

The capacitance of a capacitor of a parallel plate is given by:

[tex]C=\frac{kA\cdot\epsilon_o}{d}[/tex]

Where:

[tex]\begin{gathered} \epsilon_o=_{\text{ }}Vacuum_{\text{ }}permittivity \\ k=_{\text{ }}Dielectric_{\text{ }}constant=2 \\ A=_{\text{ }}Area_{\text{ }}of_{\text{ }}the_{\text{ }}plates \\ d=_{\text{ }}distance_{\text{ }}betwen_{\text{ }}the_{\text{ }}plates \end{gathered}[/tex]

We also know:

[tex]\begin{gathered} Q=\frac{q}{V} \\ so\colon \\ \frac{q}{V}=\frac{kA\cdot\epsilon_o}{d} \\ V=\frac{qd}{kA\cdot\epsilon_o} \\ if \\ k=2 \\ V=\frac{1}{2}(\frac{qd}{kA\cdot\epsilon_o}) \end{gathered}[/tex]

As we can see the voltage halves

And since:

[tex]\begin{gathered} E=\frac{V}{d} \\ W=qV \end{gathered}[/tex]

We can conclude that electric field and also the energy halve

Part C and D please, part A=291.2 and part B=33.8

Answers

Given,

The mass of the skater, m=63 kg

The coefficient of static friction, μs=0.4

The coefficient of the kinetic friction, μk=0.02

F₁=242 N

F₂=162 N

(c) The static friction is given by,

[tex]f_s=N\mu_s[/tex]

Where N is the normal force.

The normal force acting on the skater is

[tex]N=mg[/tex]

Where g is the acceleration due to gravity.

Therefore the static friction is given by,

[tex]f_s=mg\mu_s[/tex]

On substituting the known values,

[tex]\begin{gathered} f_s=63\times9.8\times0.4 \\ =246.96\text{ N} \end{gathered}[/tex]

Therefore the static friction on the skater is 246.96 N

d)The net force acting on the skater is

[tex]\begin{gathered} F_{\text{net}}=ma_{} \\ =F_{\text{tot}}-f \\ =F_{\text{tot}}-N\mu_k \\ =F_{\text{tot}}-mg\mu_k \end{gathered}[/tex]

On substituting the known values,

[tex]\begin{gathered} 63a=291.2-63\times9.8\times0.02 \\ a=\frac{278.85}{63} \\ =4.43m/s^2 \end{gathered}[/tex]

Thus the acceleration of the skater is 4.43 m/s²

According to Coulomb's law, the electrical force between two charged objects:A.is zero if they are opposite charges.B.increases with increasing charge.C.does not depend on the amount of charge.D.increases with increasing distance.

Answers

Answer and explanation:

A correct option is an option (B).

The electrical force between two charges is given as,

[tex]F=\frac{1}{4\pi\epsilon_0}\frac{q_1q_1}{r^2}[/tex]

The electrical force is directly proportional to the product of two charges. Thus Force will depend on two charges irrespective of their signs.

Option (A) is incorrect because if charges are opposite, the value of force will not be zero. It will be -ve.

Option (C) is incorrect because the force is directly proportional to the product of charges, it depends on the amount of charge.

Option (D) is also incorrect because the force in inversly proportional to the distance between two charges. Thus, if the distance between two charges is increased, the force between two charges will decrease.

Concllusion:

The correct option is option (B).

Find the y-component of thisvector:12.0 m73.3°Remember, angles are measured fromthe +X axis.y-component (m)

Answers

Given data:

The magnitude of the given vector is,

[tex]A=12.0\text{ m}[/tex]

An angle between the given vector and +x axis is,

[tex]\theta=73.3^o[/tex]

The formula of y-component is as follows:

[tex]A_y=A\sin \theta[/tex]

Here,

[tex]A\text{ is the magnitude of the given vector-A}[/tex]

Now, substitute known values in above equation;

[tex]\begin{gathered} A_y=12\text{ m}\times\sin 73.3^o \\ A_y=11.49\text{ m} \end{gathered}[/tex]

Therefore, the y-component of the given vector is 11.49 m

What is the area under the curve on the velocity time graph in physics represent

Answers

When given a graph of velocity vs time as the following:

The area under the curve represents the distance that the object has moved in the given time interval.

Pascal's principle states that:all liquids exert pressure downward.pressure will move toward areas where pressure is lowest.applied pressure will be transmitted throughout a fluid.None of the choices are correct.

Answers

According to Pascal's principle states that applied pressure will be transmitted throughout a fluid.

Thus, third one is the correct option.

in four hours, a hiker in a canyon goes from 892ft to 256 ft above the canyon floor. Find the hikers vertical speed.

Answers

From the given question, we can deduce the following information:

• Time, t = 4 hours

,

• The hiker goes from 892 ft to 256 ft above the canyon floor.

Let's find the vertical speed of the hiker.

To find the speed, apply the formula:

[tex]speed=\frac{dis\tan ce}{time}[/tex]

To find the distance, we have:

Distance covered = 256 ft - 892 ft = -636 ft.

Hence, to find the vertical speed, we have:

[tex]\begin{gathered} speed=\frac{dis\tan ce}{time} \\ \\ \text{speed}=\frac{-636}{4}=-159\text{ ft/h} \\ \end{gathered}[/tex]

Therefore, the vertical speed of the hiker is -159 feet per hour.

ANSWER:

What is the equation for finding the gravitational force of an object? (1 point)

Answers

The equation for finding the gravitational force of an object is F = mg.

option D is the correct answer.

What is gravitational force of an object?

Gravitational force is an attractive force that exists between all objects with mass due to the influence of the Earth's gravitational pull.

An object with mass attracts another object with mass.

The magnitude of the force is directly proportional to the masses of the two objects and inversely proportional to the square of the distance between the two objects according to Newton's law of universal gravitation.

Mathematically, the formula for gravitational force is given as;

F = mg

where;

m is the mass of the objectg is acceleration due to gravity

Learn more about gravitational force here: https://brainly.com/question/72250

#SPJ1

The fundamental frequency of a pipe is the lowest resonant frequency. Is this true or false?

Answers

By definition, the fundamental frequency of an object is the lowest frequency such that the object resonates.

Then, the answer is:

[tex]\text{True}[/tex]

Answer:

By definition, the fundamental frequency of an object is the lowest frequency such that the object resonates.

Then, the answer is: true

Explanation:

You're standing on a ramp that is inclined at a 25 degree angle. The weight (Fg) experienced is 50N. What is the parallel force (Fx) and the perpendicular force (Fy)?I need to do the following: 1. Draw a free body diagram2.Identify Givens and Unknowns3.Identify the Equations4.Set up the equation using the givens and unknowns5.Solve

Answers

Answer:

Fx = 21.13 N

Fy = 45.32 N

Explanation:

In this case, the free body diagram is:

We can see that there is a right triangle formed by the weight (Fg) and the ramp. So, we can calculate the value of angle θ.

25 + 90 + θ = 180

115 + θ = 180

θ = 180 - 115

θ = 65

Because the sum of the interior angles of a triangle is always 180 degrees.

Now, we can calculate the parallel force and the perpendicular force using the trigonometric functions sine and cosine, so

Fx = Fg cos θ

Fy = Fg sin θ

So, replacing Fg = 50 N and θ = 65, we get:

Fx = 50 cos(65) = 21.13 N

Fy = 50 sin(65) = 45.32 N

Therefore, the answers are:

Fx = 21.13 N

Fy = 45.32 N

A bicycle wheel, of radius 0.300 m and mass 2.07 kg (concentrated on the rim), is rotating at 4.00 rev/s. After 59.0 s the wheel comes to a stop because of friction. What is the magnitude of the average torque due to frictional forces?

Answers

ANSWER:

0.079 J

STEP-BY-STEP EXPLANATION:

The average torque exerted due to friction force is:

[tex]\tau=I\alpha[/tex]

Here, I is the moment of inertia and α is the angular acceleration.

The mass of wheel is concentrated at the rim. Therefore, the moment of inertia of wheel is,

[tex]I=m\cdot r^2[/tex]

Here, m is the mass of wheel and r is the radius of wheel.

replacing:

[tex]\begin{gathered} I=2.07\cdot0.3^2 \\ I=0.1863\text{ kg}\cdot s^2 \end{gathered}[/tex]

The angular speed is decreased from 4 rev/s to 0 rev/s in 59 sec. Therefore, the angular acceleration is calculated as:

[tex]\begin{gathered} \alpha=\frac{0-4}{59} \\ \alpha=0.068\cdot\frac{rev}{\sec}\cdot\frac{2\pi\text{ rad/rev}}{1rev/sec^2} \\ \alpha=-0.426rad/sec^2 \end{gathered}[/tex]

The negative sign represents that the wheel is de-accelerating.

Replacing:

[tex]\begin{gathered} \tau=I\alpha \\ \tau=0.1863\cdot-0.426 \\ \tau=-0.079\text{J} \end{gathered}[/tex]

The negative sign represents the direction of torque i.e. clockwise direction. Thus, the magnitude of average torque is 0.079 J.

URGENT!! ILL GIVE
BRAINLIEST!!!! AND 100 POINTS!!!!!!

Answers

Answer:

b heated gas will have decreased kinetic energy and decreased density

Explanation:

gas loses weight

Could really some help answering this question I don’t really understand! :)

Answers

Given:

Object A is at a higher temperature than object B.

Here, there is a difference in temperature.

Temperature flow is similar to water flow.

The water stops flowing if the water level becomes the same in botht containers.

Similarly, thermal energy transfers between the objects until the objects have changed the temperature by the same amount.

Thus, option B is correct.

What is the maximum speed at which a cyclist can move on a bend and at an angle from the vertical, he should deviate to the side of the bend, so as not to fall, if the coefficient of friction of the wheel from the road is 0.4 and the radius of curvature of the road is 100m

Answers

Given,

The coefficient of friction between the wheel and the road, μ=0.4

The radius of curvature of the road, r=100 m

The centripetal force for the cyclist to move in the curved path is provided by the friction between the road and the wheel.

Thus,

[tex]\begin{gathered} \frac{mv^2}{r}=\mu mg \\ \Rightarrow v^2=r\mu g \\ v=\sqrt[]{r\mu g} \end{gathered}[/tex]

Where m is the mass of the cyclist and the cycle and g is the acceleration due to gravity.

How much does James have to pay for a vaccum cleaner that he uses for 4 hours a week.  In James's neighborhood, the cost of electricity is 27 cents per kwhr.The power is P = 800 W

Answers

Given:

The power of the vacuum cleaner is P = 800 W = 0.8 kW

The time consumption is t = 4 hours a week.

The cost of electricity is 27 cents per kWh

To find the cost of the electricity for using the vacuum cleaner for a week.

Explanation:

First, we need to find the energy.

The energy can be calculated as

[tex]\begin{gathered} E=P\times t \\ =0.8\times4 \\ =3.2\text{ kWh} \end{gathered}[/tex]

The cost of electricity per week due to vacuum cleaner will be

[tex]27\times3.2=86.4\text{ cents}[/tex]

Final Answer: The cost of electricity James has to pay for a vacuum cleaner for using it 4 hours a week is 86.4 cents.

Victor is driving south. He is traveling at 12 m/s, when he enters an area with a new speed limit. Aver a period of 6 seconds, his speed increases from 12 m/s to 29 m/s. What is Victor’s acceleration during this period?

Answers

Answer:

1.1417 m/s²

Explanation:

Acceleration = final velocity - initial velocity/ time

a= [tex]\frac{v-u}{t}[/tex]

a= [tex]\frac{29-12}{12}[/tex]

a= [tex]\frac{17}{12}[/tex]

a= 1.417 m/s²   or [tex]\frac{17}{12}[/tex] m/s² in fraction form

John is at the park playing fetch with his cat. He throws the ball 1m south, and his cat retrieves the ball and returns it to John. John then throws the ball 3 meters, and the cat again collects the ball and returns it to John. When the cat returns for the second time, what distance has the cat traveled?6m8m2m4m

Answers

As John throws the ball 3 meters and the cat collects the ball and returns to John. Then. the distance traveled by the cat when it returns from the second time is calculated as

[tex]\begin{gathered} d=3+3 \\ =6\text{ m} \end{gathered}[/tex]

Write a rule for the sequence. 3, -3, -9, -15. A. Start with 3 and add -6 repeatedly B. Start with -6 and add 3 repeatedly C. Start with 3 and 6 repeatedly D. Start with 3 and subtract -6 repeatedly

Answers

Substract two consecutive terms of the sequence to see if there is a common difference:

[tex]\begin{gathered} (-3)-(3)=-3-3=-6 \\ (-9)-(-3)=-9+3=-6 \\ (-15)-(-9)=-15+9=-6 \end{gathered}[/tex]

As we can see, there is a common difference of -6.

Then, if a number of the sequence is given, the next one can be found by adding -6 (which is the same as subtracting 6).

Notice that the first term of the sequence is 3.

Then, the rule for the sequence is to start with 3 and add -6 repeatedly.

Therefore, the correct choice is option A) Start with 3 and add -6 repeatedly.

What happens to the strength of an electromagnet if the current in the wire is increased?

Answers

We will have the following:

What will happen is that the magnetic field will increase; since the magnetic field increases when the current in the wire of the electromagnet increases.

Calculate the torque experienced by the door due to this force using torque is equal to force times lever armLength of the lever arm = 1 mForce = 5 N

Answers

Given:

The applied force on the door is F = 5 N

The length of the lever arm is l = 1 m

Required: Torque experienced by the torque.

Explanation:

Torque is the product of force and the distance between the force applied and the rotational axis.

The force is applied on one side of the lever arm while the rotational axis will be at the other end of the lever arm.

So, the distance between the rotational axis and the force applied is the length of the arm.

Torque can be calculated by the formula

[tex]\tau=F\times l[/tex]

On substituting the values, the torque will be

[tex]\begin{gathered} \tau=5\times1 \\ =5\text{ N m } \end{gathered}[/tex]

Final Answer: The torque experienced by the door is 5 N m.

The mass of a satellite orbiting Earth is 15000 kg.

Answers

Newton's universal law of gravity

[tex]F=G\frac{m1\cdot m2}{r^2^{}}[/tex]

Where:

F= force between objects

m1= mass 1 = 15,000 kg

m2= mass 2 = 6x10^24

r = distance = 34,000,000 m

G= universal contant of gravitation = 6.67 x10^-11

Replacing:

[tex]F=6.67\cdot10^{-11}\cdot\frac{15,000\cdot6x10^{24}}{(34,000,000)^2}[/tex]

F= 5,193 N

A proton is accelerated through a potential difference of 100 V and then enters a region where it is moving perpendicular to a magnetic field B = 0.20 T. Calculate the radius of the circular path in which it will travel.

Answers

The radius of the circular path in which it will travel inside the magnetic field will be 7mm.

Magnetic field is the area around a magnet or an electric charge in which the effect of magnetism is felt.

Magnetic field of the proton = B = 0.20 T

Electric potential difference of the proton = E = 100 V

Kinetic energy of proton = 100p = [tex]\frac{1}{2}m v^{2}[/tex]

[tex]v = \sqrt{\frac{2*100e}{m} }[/tex]

The centripetal acceleration is provided by the magnetic field.

therefore,

[tex]\frac{mv^{2} }{r} = evB[/tex]

mv = eBr

[tex]r = \frac{mv}{eB}[/tex]

m = 1.67 × [tex]10^{-27}[/tex] kg

e = 1.60 ×  [tex]10^{-19}[/tex] C

B = 0.20 T

v =  [tex]\sqrt{\frac{2*100e}{m} }[/tex] = 13.8 × [tex]10^{4}[/tex] m/s

on putting the values of all the quantities in the equation of r, we get

r = 0.07 m

r = 7 mm

The radius of the circular path is 7 mm.

To know more about magnetic field,

https://brainly.com/question/15062677

#SPJ1

i need help please. i tried different answers but i cant get it.

Answers

Let's make a diagram to visualize the angle.

As you can observe, the angle is closer to the South orientation.

Therefore, the new angle is 42 degrees from South.

you have entered a 134-mile biathlon that consists of a run and a bicycle race. During your run, your average velocity is 6 miles per hour, and during your bicycle ace, your average velocity is 29 miles per hour. You finish the race in 7 hours. What is the distance of the run? What is the distance of the bicycle race?upposeWhat is the distance of the run?miles

Answers

ANSWER:

Distance of the run: 18 miles

Distance of the bicycle race: 116 miles

STEP-BY-STEP EXPLANATION:

Given:

Total distance = 134 miles

Total time = 7 hours

Average velocity during running = 6 mph

Average velocity during bicycle = 29 mph

Let x be the running distance and y be the bicycle distance.

We know that velocity equals distance in a given time, like this:

[tex]\begin{gathered} v=\frac{d}{t} \\ \\ \text{ Therefore:} \\ \\ t=\frac{d}{v} \end{gathered}[/tex]

Knowing the above, we can establish the following system of equations:

[tex]\begin{gathered} t_1+t_2=7\rightarrow\frac{d_1}{v_1}+\frac{d_2}{v_2}=7\rightarrow\frac{x}{6}+\frac{y}{29}=7\text{ \lparen1\rparen} \\ \\ x+y=134\rightarrow x=134-y\text{ \lparen2\rparen} \end{gathered}[/tex]

We substitute the second equation in the first and obtain the following:

[tex]\begin{gathered} \frac{134-y}{6}+\frac{y}{29}=7 \\ \\ \frac{(134-y)(29)+6y}{6\cdot29}=7 \\ \\ \frac{3886-29y+6y}{174}=7 \\ \\ 3886-23y=7\cdot174 \\ \\ y=\frac{1218-3886}{-23}=\frac{-2668}{-23} \\ \\ y=116\rightarrow\text{ bicycle distance} \\ \\ \text{ now, for x:} \\ \\ x=134-116 \\ \\ x=18\rightarrow\text{ running distance} \end{gathered}[/tex]

Therefore:

The distance of running is 18 miles and the distance by bicycle is 116 miles.

One way to create more fissionable fuel in the form of plutonium-239 is in which of the following?Select one:a.a coal-fire power station b.radioactive wastec.a meltdown of the cored.a breeder reactor

Answers

The correct answer is radioactive waste.

The Plutonium-239 is waste material of fuel road used in a nuclear power plant for the electricity production. The use of nuclear power plant will increase the radioactive waste and the production of Plutonum-239 will be more.

Thus, option b is

Viewers of Star Trek hear of an antimatter drive on the Starship Enterprise. One possibility for such a futuristic energy source is to store antimatter charged particles in a vacuum chamber, circulating in a magnetic field, and then extract them as needed. Antimatter annihilates with normal matter, producing pure energy. What strength (in T) magnetic field is needed to hold antiprotons, moving at 6.10 ✕ 107 m/s in a circular path 1.70 m in radius? Antiprotons have the same mass as protons but the opposite (negative) charge. (Enter the magnitude.)

Answers

Data:

B=?, v=6.10*10^7 m/s, R=1.70m, m=1.67*10^(-27)kg, q=-1.6*10^(-19)C

Answer:

Firstly, we need to remember the formula for the Magnetic force, which is:

[tex]F_m=|q|vB[/tex]

However, in this scenario the magnetic force will act as a centripetal force, thus:

[tex]F_m=\frac{mv^2}{R}[/tex]

If we equal both of them:

[tex]|q|vB=\frac{mv^2}{R}\Rightarrow B=\frac{mv}{|q|R}[/tex]

Replacing our values:

[tex]B=\frac{1.67*10^{-27}*6.1*10^7}{1.6*10^{-19}*1.7}=0.3745T[/tex]

Then, our magnetic field will be B=0.3745T

which choice are equivalent to the expression below? check all that apply. 4 square root 5answer choices: square root of 16*6, square root of 32*3, square root of 96, square of 24, 96, square root of 4*36

Answers

Given

4 square root 6

[tex]4\sqrt[]{6}[/tex]

Wich choices are equivalent to the expression below

[tex]\begin{gathered} \sqrt[]{16}\sqrt[]{6} \\ \sqrt[]{32}\sqrt[]{3} \\ \sqrt[]{96} \end{gathered}[/tex]

The first three options are the correct answers.

2. A Carnot engine operates between a 1500 C reservoir and a -20 C reservoir. What is the efficiency of the engine?1. 83%2. 86 %3. 99 %4. 1.3 %

Answers

The efficiency of a Carnot engine is given by

[tex]\eta=1-\frac{T_C}{T_H}[/tex]

Where TC is the temperature of the cold source in kelvin and TH is the temperature of the hot source in kelvin.

Let us first convert the given temperatures from celsius to kelvin.

[tex]\begin{gathered} T_C=-20+273=253K \\ T_H=1500+273=1773K \end{gathered}[/tex]

So, the efficiency is

[tex]\begin{gathered} \eta=1-\frac{253}{1773} \\ \eta=0.857 \\ \eta=85.7\% \end{gathered}[/tex]

Rounding off to a whole number

[tex]\eta=86\%[/tex]

Therefore, the efficiency of the engine is 86%

That is correct. Thank you

Circuit breakers are connected in series so that the circuit will open in case of dangerously high current.TrueFalse

Answers

Circuit breakers are connected in series so that the circuit will open in case of dangerously high current.

that is True

Other Questions
Need help with number 2 not sure how to solve it all can i have help with this please Cross a P generation, LlBb with llbb. Assume the following F1 generation:LlBb: 480Llbb: 20llBb: 20llbb: 480How far apart are L and B? Im stuck on this part and Im not sure if its just me being stressed out but I just can quiet understand. someone help pleaseeeee Assume that a researcher randomly selects 14 newborn babies and counts the number of girls, x. The probabilities corresponding to the 14 possible values of x are summarized in the given table. Answer the questions using the table.Find the probability of selecting 5 or fewer girls. Non-vascular plants are limited in size for all of the following reasons EXCEPT theyA)have no xylem to carry water.B)have no phloem to store sugars.C)have tiny seeds that can only travel short distances.D)must have moisture for their sperm to swim to the egg of other plants. Your 90-year-old grandmother had a stroke and has been in a nursing home for about 3 months. On your last visit you noticed a red, blistered area on the back of her leg. Name your grandmother's skin condition, its stage of development, and the proper treatment. If v, = (2,4) and v2 = (-1,5), then w,-V is equal to which of the following?O 18O (-2. 20)O 22O (8.-5) Question 25 of 27Which of the following is an example of independent events?OA. Rolling a 6 on a number cube and spinning a 7 on a spinnerB. Drawing a jack from a standard deck of cards and then drawing a2 without replacing the jackC. Selecting a green marble from a bag of 10 different coloredmarbles and then selecting a second green marble without puttingthe first backOD. Owning cows and living on a dairy farm List three of the rules of interpretation the student of the Bible must follow. A chemist wishes to mix a solution that is 6% acid. She has on hand 14 liters of a 4% acid solution and wishes to add some 10% acid solution to obtain the desired 6% acid solution. How much 10% acid solution should she add? The vertices of a rectangle are R(-5, -5), S(-1, -5), T(-1, 1), and U(-5, 1). After translation, R' is the point (-9, 1). Find the translation rule andcoordinates of U'.2 of 27 Place the numbers in order fromgreatest to least27, 62%, -0.062,65,0. Question 6 of 13Use a calculator to find the correlation coefficient of the data set.X13LO5y2014106916 4 Mr mustard bought a bag of citrus tree fertilizer. He used 1/5 of the bag for his orange tree and 2/3 of the bag for his lemon tree. About how much of the fertilizer does he have left?A. 0.5 B. 0.8 C. 0 D. 0.1 Compltez la phrase en choisissant les vtements qui correspondent l'image Thomas porte souvent____bleue avec_____noir et________noires. 1. un pull 1. un pantalon 1. des sandales 2. un tee-shirt 2.un jean 2. des bottes 3. un chemise 3. un short 3. des chaussures Can I please just get the answer Im trying to get this class done before the 10th One year, the population of a city was 253,000. Several years later it was 288,420. Find the percent increase. Which two major parts of the earth system does this photo most clearly show If 250 ml of methane, ch4, effuses through a small hole in 28 s, the time required for the same volume of helium to pass through the hole under the same conditions will be.